Machscher Kegel

Online-Sprechstunde für Physik für Ingenieure (AIW) - betreut vom Institut "Optische und Elektronische Materialien" (WiSe)

Moderatoren: SlawaL, LukasM, SebastianJ, (M) Mod.-Team Allgemein

Antworten
Benutzeravatar
JHW
Uni-Mitarbeiter
Uni-Mitarbeiter
Beiträge: 104
Registriert: Mo, 03. Jul. 06, 20:30
Kontaktdaten:

Machscher Kegel

Beitrag von JHW » Mo, 04. Feb. 08, 09:03

Folgende Frage erreichte mich über Email:
ich hab ne frage zum Machschen Kegel. ist der Winkel im Skript auf S.192 richtig eingezeichnet? wenn ja kann ich mir die Formel nicht erklären, da dann der Eintritt in die Schallgeschwindigkeit schon bei einem Winkel von 90° sein würde.
Der Winkel [tex]\alpha[/tex] ist richtig eingezeichnet und befindet sich in dem rechtwinkligen Dreieck, dass durch die Punkte ABQ beschrieben wird. Demnach gilt für [tex]\alpha[/tex]:
[tex]\alpha=\frac{|AB|}{|AQ|}=\frac{ct}{v_Qt}[/tex].

Wenn jetzt die Quelle sich mit exakt Schallgeschwindigkeit bewegt ist [tex]\alpha=90^o[/tex] und der Kegel degradiert zu einer ebenen Wellenfront.

Antworten